Proving a limit exists using delta and epsilon?












0












$begingroup$


First time posting, I did calculations, would someone be so kind to see if i'm right or wrong, i'm a part-time student so i do not have a lecturer etc.



f (x,y) = x2/ √(x2+ y2)



Prove from First principles
Lim f(x,y) = 0
(x,y) -> (0,0)



Answer:
The domain Df ∈ {(x,y) ∈ R}



0 < ll√[(x - 0)2 + (y - 0) 2]ll < 𝛿 ( from definition ll√[(x - a)2 + (y - b) 2]ll < 𝛿 )
0 < ll√[(x )2 + (y) 2]ll < 𝛿
0 < ll√[(x )2 + (y) 2]ll < 1 (where 1 < 𝛿)



lx-al = lxl = ll√[(x )2 + (y ) 2]ll = {(x,y) – (0,0)}
ly-bl = lyl = ll√[(x )2 + (y ) 2]ll = {(x,y) – (0,0)}



lf(x) - L l < ε
lx2/ √(x2+ y2) – 0 l < ε
lx2/ √(x2+ y2) – 0 l < ε
lx2l/l √(x2+ y2)l < ε

lxl 2/ √(x2+ y2) < ε



{√(x2+ y2)}2/ √(x2+ y2) < ε (substituted in lxl2 = ll√[(x )2 + (y ) 2]ll
√(x2+ y2) < ε



If 0 < ll√[(x )2 + (y) 2]ll < 1, where 1 < 𝛿, then lf(x) - L l < ε for Lim f(x,y) = 0
(x,y)->(0,0)










share|cite|improve this question











$endgroup$








  • 3




    $begingroup$
    Hi, here is a basic tutorial. Please type math using MathJax. Gate.
    $endgroup$
    – xbh
    Jan 23 at 6:13










  • $begingroup$
    Thanks going there now
    $endgroup$
    – Shaun Weinberg
    Jan 23 at 6:24
















0












$begingroup$


First time posting, I did calculations, would someone be so kind to see if i'm right or wrong, i'm a part-time student so i do not have a lecturer etc.



f (x,y) = x2/ √(x2+ y2)



Prove from First principles
Lim f(x,y) = 0
(x,y) -> (0,0)



Answer:
The domain Df ∈ {(x,y) ∈ R}



0 < ll√[(x - 0)2 + (y - 0) 2]ll < 𝛿 ( from definition ll√[(x - a)2 + (y - b) 2]ll < 𝛿 )
0 < ll√[(x )2 + (y) 2]ll < 𝛿
0 < ll√[(x )2 + (y) 2]ll < 1 (where 1 < 𝛿)



lx-al = lxl = ll√[(x )2 + (y ) 2]ll = {(x,y) – (0,0)}
ly-bl = lyl = ll√[(x )2 + (y ) 2]ll = {(x,y) – (0,0)}



lf(x) - L l < ε
lx2/ √(x2+ y2) – 0 l < ε
lx2/ √(x2+ y2) – 0 l < ε
lx2l/l √(x2+ y2)l < ε

lxl 2/ √(x2+ y2) < ε



{√(x2+ y2)}2/ √(x2+ y2) < ε (substituted in lxl2 = ll√[(x )2 + (y ) 2]ll
√(x2+ y2) < ε



If 0 < ll√[(x )2 + (y) 2]ll < 1, where 1 < 𝛿, then lf(x) - L l < ε for Lim f(x,y) = 0
(x,y)->(0,0)










share|cite|improve this question











$endgroup$








  • 3




    $begingroup$
    Hi, here is a basic tutorial. Please type math using MathJax. Gate.
    $endgroup$
    – xbh
    Jan 23 at 6:13










  • $begingroup$
    Thanks going there now
    $endgroup$
    – Shaun Weinberg
    Jan 23 at 6:24














0












0








0





$begingroup$


First time posting, I did calculations, would someone be so kind to see if i'm right or wrong, i'm a part-time student so i do not have a lecturer etc.



f (x,y) = x2/ √(x2+ y2)



Prove from First principles
Lim f(x,y) = 0
(x,y) -> (0,0)



Answer:
The domain Df ∈ {(x,y) ∈ R}



0 < ll√[(x - 0)2 + (y - 0) 2]ll < 𝛿 ( from definition ll√[(x - a)2 + (y - b) 2]ll < 𝛿 )
0 < ll√[(x )2 + (y) 2]ll < 𝛿
0 < ll√[(x )2 + (y) 2]ll < 1 (where 1 < 𝛿)



lx-al = lxl = ll√[(x )2 + (y ) 2]ll = {(x,y) – (0,0)}
ly-bl = lyl = ll√[(x )2 + (y ) 2]ll = {(x,y) – (0,0)}



lf(x) - L l < ε
lx2/ √(x2+ y2) – 0 l < ε
lx2/ √(x2+ y2) – 0 l < ε
lx2l/l √(x2+ y2)l < ε

lxl 2/ √(x2+ y2) < ε



{√(x2+ y2)}2/ √(x2+ y2) < ε (substituted in lxl2 = ll√[(x )2 + (y ) 2]ll
√(x2+ y2) < ε



If 0 < ll√[(x )2 + (y) 2]ll < 1, where 1 < 𝛿, then lf(x) - L l < ε for Lim f(x,y) = 0
(x,y)->(0,0)










share|cite|improve this question











$endgroup$




First time posting, I did calculations, would someone be so kind to see if i'm right or wrong, i'm a part-time student so i do not have a lecturer etc.



f (x,y) = x2/ √(x2+ y2)



Prove from First principles
Lim f(x,y) = 0
(x,y) -> (0,0)



Answer:
The domain Df ∈ {(x,y) ∈ R}



0 < ll√[(x - 0)2 + (y - 0) 2]ll < 𝛿 ( from definition ll√[(x - a)2 + (y - b) 2]ll < 𝛿 )
0 < ll√[(x )2 + (y) 2]ll < 𝛿
0 < ll√[(x )2 + (y) 2]ll < 1 (where 1 < 𝛿)



lx-al = lxl = ll√[(x )2 + (y ) 2]ll = {(x,y) – (0,0)}
ly-bl = lyl = ll√[(x )2 + (y ) 2]ll = {(x,y) – (0,0)}



lf(x) - L l < ε
lx2/ √(x2+ y2) – 0 l < ε
lx2/ √(x2+ y2) – 0 l < ε
lx2l/l √(x2+ y2)l < ε

lxl 2/ √(x2+ y2) < ε



{√(x2+ y2)}2/ √(x2+ y2) < ε (substituted in lxl2 = ll√[(x )2 + (y ) 2]ll
√(x2+ y2) < ε



If 0 < ll√[(x )2 + (y) 2]ll < 1, where 1 < 𝛿, then lf(x) - L l < ε for Lim f(x,y) = 0
(x,y)->(0,0)







limits






share|cite|improve this question















share|cite|improve this question













share|cite|improve this question




share|cite|improve this question








edited Jan 23 at 7:46







Shaun Weinberg

















asked Jan 23 at 6:10









Shaun WeinbergShaun Weinberg

62




62








  • 3




    $begingroup$
    Hi, here is a basic tutorial. Please type math using MathJax. Gate.
    $endgroup$
    – xbh
    Jan 23 at 6:13










  • $begingroup$
    Thanks going there now
    $endgroup$
    – Shaun Weinberg
    Jan 23 at 6:24














  • 3




    $begingroup$
    Hi, here is a basic tutorial. Please type math using MathJax. Gate.
    $endgroup$
    – xbh
    Jan 23 at 6:13










  • $begingroup$
    Thanks going there now
    $endgroup$
    – Shaun Weinberg
    Jan 23 at 6:24








3




3




$begingroup$
Hi, here is a basic tutorial. Please type math using MathJax. Gate.
$endgroup$
– xbh
Jan 23 at 6:13




$begingroup$
Hi, here is a basic tutorial. Please type math using MathJax. Gate.
$endgroup$
– xbh
Jan 23 at 6:13












$begingroup$
Thanks going there now
$endgroup$
– Shaun Weinberg
Jan 23 at 6:24




$begingroup$
Thanks going there now
$endgroup$
– Shaun Weinberg
Jan 23 at 6:24










1 Answer
1






active

oldest

votes


















1












$begingroup$

I will suggest a much simpler argument. $delta =epsilon$ works because $x^{2} leq {x^{2}+y^{2}}$, so $|f(x,y)| leq sqrt {x^{2}+y^{2}}$. Hence $sqrt {x^{2}+y^{2}} <epsilon$ implies $|f(x,y)| <epsilon$.






share|cite|improve this answer











$endgroup$













  • $begingroup$
    Thank you, i have updated the question i am still struggling with this sites way of doing things. Could you kindly explain?
    $endgroup$
    – Shaun Weinberg
    Jan 23 at 7:47










  • $begingroup$
    @ShaunWeinberg Sorry, I find it very hard to read what you have written. There is nothing wrong in doing some simple manipulations before coming up with a $delta$, so my answer is usually accepted by any teacher.
    $endgroup$
    – Kavi Rama Murthy
    Jan 23 at 7:50













Your Answer





StackExchange.ifUsing("editor", function () {
return StackExchange.using("mathjaxEditing", function () {
StackExchange.MarkdownEditor.creationCallbacks.add(function (editor, postfix) {
StackExchange.mathjaxEditing.prepareWmdForMathJax(editor, postfix, [["$", "$"], ["\\(","\\)"]]);
});
});
}, "mathjax-editing");

StackExchange.ready(function() {
var channelOptions = {
tags: "".split(" "),
id: "69"
};
initTagRenderer("".split(" "), "".split(" "), channelOptions);

StackExchange.using("externalEditor", function() {
// Have to fire editor after snippets, if snippets enabled
if (StackExchange.settings.snippets.snippetsEnabled) {
StackExchange.using("snippets", function() {
createEditor();
});
}
else {
createEditor();
}
});

function createEditor() {
StackExchange.prepareEditor({
heartbeatType: 'answer',
autoActivateHeartbeat: false,
convertImagesToLinks: true,
noModals: true,
showLowRepImageUploadWarning: true,
reputationToPostImages: 10,
bindNavPrevention: true,
postfix: "",
imageUploader: {
brandingHtml: "Powered by u003ca class="icon-imgur-white" href="https://imgur.com/"u003eu003c/au003e",
contentPolicyHtml: "User contributions licensed under u003ca href="https://creativecommons.org/licenses/by-sa/3.0/"u003ecc by-sa 3.0 with attribution requiredu003c/au003e u003ca href="https://stackoverflow.com/legal/content-policy"u003e(content policy)u003c/au003e",
allowUrls: true
},
noCode: true, onDemand: true,
discardSelector: ".discard-answer"
,immediatelyShowMarkdownHelp:true
});


}
});














draft saved

draft discarded


















StackExchange.ready(
function () {
StackExchange.openid.initPostLogin('.new-post-login', 'https%3a%2f%2fmath.stackexchange.com%2fquestions%2f3084143%2fproving-a-limit-exists-using-delta-and-epsilon%23new-answer', 'question_page');
}
);

Post as a guest















Required, but never shown

























1 Answer
1






active

oldest

votes








1 Answer
1






active

oldest

votes









active

oldest

votes






active

oldest

votes









1












$begingroup$

I will suggest a much simpler argument. $delta =epsilon$ works because $x^{2} leq {x^{2}+y^{2}}$, so $|f(x,y)| leq sqrt {x^{2}+y^{2}}$. Hence $sqrt {x^{2}+y^{2}} <epsilon$ implies $|f(x,y)| <epsilon$.






share|cite|improve this answer











$endgroup$













  • $begingroup$
    Thank you, i have updated the question i am still struggling with this sites way of doing things. Could you kindly explain?
    $endgroup$
    – Shaun Weinberg
    Jan 23 at 7:47










  • $begingroup$
    @ShaunWeinberg Sorry, I find it very hard to read what you have written. There is nothing wrong in doing some simple manipulations before coming up with a $delta$, so my answer is usually accepted by any teacher.
    $endgroup$
    – Kavi Rama Murthy
    Jan 23 at 7:50


















1












$begingroup$

I will suggest a much simpler argument. $delta =epsilon$ works because $x^{2} leq {x^{2}+y^{2}}$, so $|f(x,y)| leq sqrt {x^{2}+y^{2}}$. Hence $sqrt {x^{2}+y^{2}} <epsilon$ implies $|f(x,y)| <epsilon$.






share|cite|improve this answer











$endgroup$













  • $begingroup$
    Thank you, i have updated the question i am still struggling with this sites way of doing things. Could you kindly explain?
    $endgroup$
    – Shaun Weinberg
    Jan 23 at 7:47










  • $begingroup$
    @ShaunWeinberg Sorry, I find it very hard to read what you have written. There is nothing wrong in doing some simple manipulations before coming up with a $delta$, so my answer is usually accepted by any teacher.
    $endgroup$
    – Kavi Rama Murthy
    Jan 23 at 7:50
















1












1








1





$begingroup$

I will suggest a much simpler argument. $delta =epsilon$ works because $x^{2} leq {x^{2}+y^{2}}$, so $|f(x,y)| leq sqrt {x^{2}+y^{2}}$. Hence $sqrt {x^{2}+y^{2}} <epsilon$ implies $|f(x,y)| <epsilon$.






share|cite|improve this answer











$endgroup$



I will suggest a much simpler argument. $delta =epsilon$ works because $x^{2} leq {x^{2}+y^{2}}$, so $|f(x,y)| leq sqrt {x^{2}+y^{2}}$. Hence $sqrt {x^{2}+y^{2}} <epsilon$ implies $|f(x,y)| <epsilon$.







share|cite|improve this answer














share|cite|improve this answer



share|cite|improve this answer








edited Jan 23 at 6:44

























answered Jan 23 at 6:39









Kavi Rama MurthyKavi Rama Murthy

63.9k42464




63.9k42464












  • $begingroup$
    Thank you, i have updated the question i am still struggling with this sites way of doing things. Could you kindly explain?
    $endgroup$
    – Shaun Weinberg
    Jan 23 at 7:47










  • $begingroup$
    @ShaunWeinberg Sorry, I find it very hard to read what you have written. There is nothing wrong in doing some simple manipulations before coming up with a $delta$, so my answer is usually accepted by any teacher.
    $endgroup$
    – Kavi Rama Murthy
    Jan 23 at 7:50




















  • $begingroup$
    Thank you, i have updated the question i am still struggling with this sites way of doing things. Could you kindly explain?
    $endgroup$
    – Shaun Weinberg
    Jan 23 at 7:47










  • $begingroup$
    @ShaunWeinberg Sorry, I find it very hard to read what you have written. There is nothing wrong in doing some simple manipulations before coming up with a $delta$, so my answer is usually accepted by any teacher.
    $endgroup$
    – Kavi Rama Murthy
    Jan 23 at 7:50


















$begingroup$
Thank you, i have updated the question i am still struggling with this sites way of doing things. Could you kindly explain?
$endgroup$
– Shaun Weinberg
Jan 23 at 7:47




$begingroup$
Thank you, i have updated the question i am still struggling with this sites way of doing things. Could you kindly explain?
$endgroup$
– Shaun Weinberg
Jan 23 at 7:47












$begingroup$
@ShaunWeinberg Sorry, I find it very hard to read what you have written. There is nothing wrong in doing some simple manipulations before coming up with a $delta$, so my answer is usually accepted by any teacher.
$endgroup$
– Kavi Rama Murthy
Jan 23 at 7:50






$begingroup$
@ShaunWeinberg Sorry, I find it very hard to read what you have written. There is nothing wrong in doing some simple manipulations before coming up with a $delta$, so my answer is usually accepted by any teacher.
$endgroup$
– Kavi Rama Murthy
Jan 23 at 7:50




















draft saved

draft discarded




















































Thanks for contributing an answer to Mathematics Stack Exchange!


  • Please be sure to answer the question. Provide details and share your research!

But avoid



  • Asking for help, clarification, or responding to other answers.

  • Making statements based on opinion; back them up with references or personal experience.


Use MathJax to format equations. MathJax reference.


To learn more, see our tips on writing great answers.




draft saved


draft discarded














StackExchange.ready(
function () {
StackExchange.openid.initPostLogin('.new-post-login', 'https%3a%2f%2fmath.stackexchange.com%2fquestions%2f3084143%2fproving-a-limit-exists-using-delta-and-epsilon%23new-answer', 'question_page');
}
);

Post as a guest















Required, but never shown





















































Required, but never shown














Required, but never shown












Required, but never shown







Required, but never shown

































Required, but never shown














Required, but never shown












Required, but never shown







Required, but never shown







Popular posts from this blog

Mario Kart Wii

What does “Dominus providebit” mean?

Antonio Litta Visconti Arese